Optimization: Maximizing Profit

  • Thread starter Thread starter molly16
  • Start date Start date
  • Tags Tags
    Optimization
Click For Summary
The discussion focuses on maximizing profit for a computer manufacturer based on their sales and production costs. The initial attempt incorrectly defined revenue as a function of a single unit instead of multiplying the price by the number of units sold. After clarification, the correct revenue formula was established as R = (1000x)(2000 - 5x), leading to the correct profit equation. By taking the derivative and solving, the optimal sales level was determined to be approximately 19,704 units, matching the book's answer. This highlights the importance of accurately interpreting the problem statement in optimization scenarios.
molly16
Messages
16
Reaction score
0

Homework Statement



Through market research, a computer manufacturer found that x thousand
units of its new laptop will sell at a price of 2000 - 5x dollars per unit.
The cost, C, in dollars of producing this many units is
C(x) = 15 000 000 +1 800 000x + 75x^2. Determine the level of sales
that will maximize profit.

Homework Equations


Profit = Revenue - Cost

The Attempt at a Solution



I said Revenue = 2000 - 5x and Cost = 15 000 000 +1 800 000x + 75x^2
Using the formula Profit = Revenue - Cost I subtracted them from each other and got:

Profit = -14998000 - 1800005x - 75x^2

Then I found the derivative which came out to be:

P'(x) = -1800005 - 150x

Then I set it equal to zero and solved for x:

0 = -1800005 - 150x
1800005 = -150x
x = -12000.03

but the answer in the back of the book is 19 704 units.
Can anyone explain what I did wrong?
 
Physics news on Phys.org
Welcome to PF, molly16! :smile:

Your revenue looks to be the price of only 1 unit.
Shouldn't more units be sold?
 
I like Serena said:
Welcome to PF, molly16! :smile:

Your revenue looks to be the price of only 1 unit.
Shouldn't more units be sold?

So does that mean I should multiply 2000 - 5x by x to represent the number of units?
 
I believe your problem statement says "x thousand units" that each sell at a price of "2000 - 5x" per unit.
 
I like Serena said:
I believe your problem statement says "x thousand units" that each sell at a price of "2000 - 5x" per unit.

So:
R = (1000x)(2000 - 5x)

then

Profit = Revenue - Cost
P = (1000x)(2000 - 5x) - 15 000 000 +1 800 000x + 75x^2

and

P' = 200000 - 10150x
x= 19.704

#of units = (1000x) = (1000)(19.704) = 19704 units

which was the answer in the back of the book

Thanks for the help!
 
You're welcome. :smile:
 
Question: A clock's minute hand has length 4 and its hour hand has length 3. What is the distance between the tips at the moment when it is increasing most rapidly?(Putnam Exam Question) Answer: Making assumption that both the hands moves at constant angular velocities, the answer is ## \sqrt{7} .## But don't you think this assumption is somewhat doubtful and wrong?

Similar threads

  • · Replies 4 ·
Replies
4
Views
2K
Replies
4
Views
2K
Replies
2
Views
1K
  • · Replies 1 ·
Replies
1
Views
2K
  • · Replies 9 ·
Replies
9
Views
3K
  • · Replies 2 ·
Replies
2
Views
2K
  • · Replies 1 ·
Replies
1
Views
11K
  • · Replies 4 ·
Replies
4
Views
2K
  • · Replies 3 ·
Replies
3
Views
4K
  • · Replies 25 ·
Replies
25
Views
4K